LSAT and Law School Admissions Forum

Get expert LSAT preparation and law school admissions advice from PowerScore Test Preparation.

 Administrator
PowerScore Staff
  • PowerScore Staff
  • Posts: 8917
  • Joined: Feb 02, 2011
|
#35037
Complete Question Explanation

Must Be True. The correct answer choice is (B)

In this stimulus, the government health service and PharmCo, a drug manufacturer, appear to be at
odds. The health service’s position is that if PharmCo does not provide detailed information about
the cost-effectiveness of Antinfia, an influenza medication, then it will not pay for patients to take it.
PharmCo’s response is that it will not be able to provide the data the government wants unless the
health service pays for Antinfia, because the massive clinical trials required to obtain the data cannot
happen until the drug is in widespread circulation, which can only happen if the government pays for
patients to take it.

Although the question stem is worded strangely, this is a Must Be True question. However, notice
that the reason the language in the question stem seems strange is that it announces a weaker
standard of proof than we normally have for Must Be True questions. This means that the correct
answer choice may contain language that we may typically consider inappropriate for a Must Be
True question. Given the conditional language of the positions taken by both the government service
and PharmCo, we can infer that the government health service will not pay for patients to take the
influenza medicine. This is because the health service’s requirement and PharmCo’s requirement are
incompatible. They both are demanding that the other take the first step.

Answer choice (A): This answer choice is incorrect because it is overly broad. The facts in the
stimulus dealt only with Antinfia, so we cannot make any inferences about what the government
health service does generally.

Answer choice (B): This is the correct answer choice, because we were able to infer that the
government will not pay to put Antinfia into widespread circulation, and PharmCo’s statement told
us that the only way for the drug to be in widespread circulation is for the government to pay for it.

Answer choice (C): There is no indication from the stimulus that the patients will pay for the
Antinfia themselves. Although the stimulus mentioned patients, it made no mention of their
willingness or ability to pay for the drug. To the contrary, PharmCo told us that the drug will go into
widespread circulation only if the government pays for it.

Answer choice (D): The operative word in this answer choice is should. The use of “should” implies
the application of a rule to the facts. Since the stimulus did not contain a rule, we cannot conclude
that the government should pay for patients to take the drug.

Answer choice (E): The issue in the stimulus was who was going to pay for the widespread
distribution of the Antinfia, which was required before we could answer the question of the drug’s
cost-effectiveness. Since the widespread distribution will not occur, we cannot say whether or not the
drug is cost-effective.
 srcline@noctrl.edu
  • Posts: 243
  • Joined: Oct 16, 2015
|
#30333
Hello

So after reviewing,this is most strongly supported question stem correct? So there seems to be many conditionals in this stimulus, and I don't think that I diagrammed this right:

1.PharmaCo details about Anifa Cost effectiveness :arrow: Govt pay patients medication

2. Drug widespread circulation :arrow: no trials

3. wide spread circulation :arrow: Govt. pays for Anifa

So I am not seeking a linkage in any of these statements. Can someone please explain this question, and what the best way is to approach these questions are, I'm pretty sure by this point I was running out of time.

Thankyou
Sarah
 Adam Tyson
PowerScore Staff
  • PowerScore Staff
  • Posts: 5153
  • Joined: Apr 14, 2011
|
#30391
Thanks for asking, Sarah. Let's see if we can break this down and see where things went awry!

First sentence - we have an "until" in there, and that is one of our special necessary indicators. That sentence should be diagrammed something like this:

Gov Pays --> Pharm provides details about cost effectiveness

Next sentence, we continue that chain, because "requires" is another necessary indicator. Now we add this:

Pharm provides details about cost effectiveness --> massive clinical trials

Third sentence has two conditional phrases, so let's take them one at a time. First one has another until, requiring that special approach again. That phrase looks like this:

massive clinical trials --> widespread circulation

Last phrase has the necessary indicator "only if", and looks like this:

widespread circulation --> Gov pays

We are back where we started! Link those all up and you get this long chain:

Gov Pays --> Pharm provides details about cost effectiveness --> massive clinical trials --> widespread circulation --> Gov pays

Compare this to yours, and it looks like you have some stuff going right (your first one leaps over a couple steps in the chain, but it's true, and your last one matches my last one) and some stuff going wrong (your second one looks like we got a negation wrong and maybe reversed the order - spend some time with that and let's see how you got there).

Reviewing this chain, we can see that we have a catch-22 going on here - if the Gov pays, the gov pays, but that requires a bunch of other stuff first, which can't happen until the Gov pays. The Gov won't pay until this stuff happens, and this stuff won't happen until the Gov pays. We're stuck - nothing can ever get done. That's why B is the correct answer here - we will never get to a point where any of this stuff happens, because nobody is willing or able to go first and start the chain of events moving!

I hope that clears things up. Do take a look at the conditional statements again and see if you can figure out how you got the diagrams that you did, and how you might fix them. Study up on the special cases of "unless, except, until and without" as necessary condition indicators with a twist, what we call the "Unless EquationTM".

Good luck!
 srcline@noctrl.edu
  • Posts: 243
  • Joined: Oct 16, 2015
|
#30449
Hello Adam


Thankyou for your explanation, I really got tripped up with this question with the word "until" and if it was a necessary condition or not. Your diagram clears up alot of my confusion.

Sarah
 Clay Cooper
PowerScore Staff
  • PowerScore Staff
  • Posts: 241
  • Joined: Jul 03, 2015
|
#30470
Thanks for the follow up and the feedback Sarah! It's really helpful to us to know when an explanation has done the job.

Keep working hard!
 Kellyg
  • Posts: 14
  • Joined: Jan 23, 2018
|
#46029
Did anyone else think this answer was just too easy? I feel like we've been taught/I've assumed an answer that basically gives up on the information given in the stimulus would be incorrect.
 Adam Tyson
PowerScore Staff
  • PowerScore Staff
  • Posts: 5153
  • Joined: Apr 14, 2011
|
#46646
This certainly was a low difficulty question, kellyg, at least according to the stats we have. 78% of students taking that test got it right, which puts it in our "Level 2" difficulty category and almost into Level 1. Not sure what you mean about "gives up on the information given in the stimulus" - could you elaborate on that? What we have here is a series of premises which, when put together, shows us an impossible situation, a classic Catch-22. You can't have the funding until you've had the trials, and you can't have the trials until you're in widespread circulation, which cannot happen until you've got the funding. What must be true? You'll never get the funding, never be in widespread circulation, and never have the trials.

Tell us more about your concerns about this stimulus and what you didn't like about that answer choice, and we'll see if we can shed a little light on it for you.
 Leela
  • Posts: 63
  • Joined: Apr 13, 2019
|
#64924
I tend to struggle with Must be True questions that allow for inferences.

Just to be clear, when a question stem uses "most likely to be true" and similar language, I can bring in necessary assumptions but nothing else? If so, does this allow for the Assumption Negation Technique to be used?

My automatic reaction to Must be True answer choices is the Fact Test, but clearly that would rule out all of the answer choices in this case. I'm trying to figure out where to draw the line in how much leeway is allowed, as far as bringing in information not enumerated in the stimulus, when must be true becomes most likely to be true.
 Jeremy Press
PowerScore Staff
  • PowerScore Staff
  • Posts: 1000
  • Joined: Jun 12, 2017
|
#66843
Hi Leela,

Be careful with the language "bring in." On any Must Be True question (whether "Must Be True," or the slightly less demanding "Most Strongly Supported/Most Likely To Be True"), you will not be asked to "bring in" any information beyond what is already contained in, or implied by, the stimulus. Now it's true that in almost every case the correct answer will not be explicitly stated in the stimulus. But the correct answer "necessarily follows from" the information in the stimulus.

Even though there is a soft overlap between Must Be True and Assumption questions (in that the standard for both is "necessity"), you should not apply the Assumption Negation Technique on any form of Must Be True question (whether "Must Be True" or "Most Likely To Be True" or "Most Strongly Supported"). That technique is designed specifically for Assumption questions, and it will not work on Must Be True questions.

The Fact Test does not rule out answer choice B on this question, though seeing how answer choice B satisfies the Fact Test is a little more complicated than usual. Here, answer choice B comes from the combination of facts that Adam nicely summarized in his post:
Adam Tyson wrote:You can't have the funding until you've had the trials, and you can't have the trials until you're in widespread circulation, which cannot happen until you've got the funding. What must be true? You'll never get the funding, never be in widespread circulation, and never have the trials.
Since the facts Adam rehearsed, when combined, lead inexorably to the claim in answer choice B, that answer choice satisfies the Fact Test. Notice that, in a purely mechanical way, you can compare the "content" of answer choice B against the stimulus. Antinfia is the subject of nearly every sentence of the stimulus (thus supported). The drug's being "in widespread circulation" is a subject explicitly mentioned in the second-last line of the stimulus (thus supported). The only element of answer choice B not explicitly mentioned in the stimulus is the verb "will never be," connecting Antinfia to widespread circulation. However, as Adam demonstrated, that prediction is supported by the Catch-22 nature of the facts.

In short, the only "leeway" you have with Must Be True question is for claims that necessarily follow from the facts in the stimulus. But even those claims must be supported by combinations of information or implications of single claims.

I hope this helps!

Jeremy
User avatar
 SGD2021
  • Posts: 72
  • Joined: Nov 01, 2021
|
#92817
Hello, is it always true that when we have two statements like this which we call mistaken reversals of each other (Pay-->info on cost effectiveness and info on cost effectiveness-->pay) then the thing in question won't happen?

Get the most out of your LSAT Prep Plus subscription.

Analyze and track your performance with our Testing and Analytics Package.